Select the correct answer. What is -16 + 49 written as a complex number in the form ? A. 7 + 4i B.7 - 4i C.-4+7i D.4+7i

Answers

Answer 1

Answer:

A. 7 + 4i

Step-by-step explanation:

Select the correct answer. What is -16 + 49 written as a complex number in the form ? A. 7 + 4i B.7 - 4i C.-4+7i D.4+7i

Writing as a complex number, we find the square root of -16 + 49

Hence:

√ -16 + 49 = √-16 + √49

= 4i + 7

Rearranging

= 7 + 4i

Therefore, the correct option is A. 7 + 4i


Related Questions

please help! 10 points, thank you

Answers

Answer:

261.5 ft^2

Step-by-step explanation:

Surface Area of a rectangular prism = Length x Width x Height

= 8.5ft x 6ft x 5.5ft

= 261.5 ft

A teacher writes four numbers on the board as shown blow
1
0
4
3
She asked the students to form 4-digit number, and subtract from 4310.
Write two possible solutions​

Answers

Answer:

909 and 3267

Step-by-step explanation:

There are a total of 24 possible combinations using these 4 digits. This means that any one of these combinations subtracted from 4310 would be a possible solution. Therefore, we can create the following 2 combinations (below in bold) and subtract them from 4310 to get two possible solutions...

4310 - 3401 = 909

4310 - 1043 = 3267

Finally, we have two possible solutions which would be 909 and 3267

El profesor Héctor y sus alumnos organizaron una excursión a
la Ciudad de México. Visitarán el Centro Histórico, el Castillo de
Chapultepec y el Museo de Antropologia. El costo del transporte
por alumno es de $310.75 y no incluye alimentos.
a) Para pagar el transporte, el profesor Héctor tiene que juntar
el dinero de los 37 alumnos que participaran en la excursion
¿Cuánto dinero debe reunir?​

Answers

Answer:

$8.39 por pe rsona

Step-by-step explanation:

A company sells widgets. The amount of profit, y, made by the company, is related to
the selling price of each widget, x, by the given equation. Using this equation, find out
what price the widgets should be sold for, to the nearest cent, for the company to
make the maximum profit.
y = -x2 + 72x – 458

Answers

Answer:

In order to make the maximum profit, the company should sell each widget at (the currency isn't given, so I'll be assuming USD) $36.

Step-by-step explanation:

The amount of profit y made by the company from selling each widget at x price is modeled by the equation:

[tex]y=-x^2+72x-458[/tex]

Since this is a quadratic with a negative leading coefficient, its maximum will occur at its vertex point. We want to find the price that the widget should be sold to make the maximum profit. So, we want to find the x-coordinate of the vertex. This is given by the formula:

[tex]\displaystyle x=-\frac{b}{2a}[/tex]

In this case, a = -1, b = 72, and c = -458. Substitute and evaluate:

[tex]\displaystyle x=-\frac{72}{2(-1)}=36[/tex]

In order to make the maximum profit, the company should sell each widget at (the currency isn't given, so I'll be assuming USD) $36.

A caterer charges a setup fee of $50, plus $20 per person How much will the caterer charge if 35 people attend the party, and the customer has a coupon for $100 discount off the total?

Answers

They will charge $165 for each person
The answer is 650

20x35= 700

700+50 = 750

750-100= 650

Perez throws a stone on the pond. The path traveled by the stone can be modeled by y = -2x2 + 8x + 5, where x represents the time (in seconds) and y represents the height of the stone (in feet). What is the maximum height that the stone reaches

Answers

Answer:

The maximum height that the stone reaches is of 26 feet.

Step-by-step explanation:

Vertex of a quadratic function:

Suppose we have a quadratic function in the following format:

[tex]f(x) = ax^{2} + bx + c[/tex]

It's vertex is the point [tex](x_{v}, y_{v})[/tex]

In which

[tex]x_{v} = -\frac{b}{2a}[/tex]

[tex]y_{v} = -\frac{\Delta}{4a}[/tex]

Where

[tex]\Delta = b^2-4ac[/tex]

If a<0, the vertex is a maximum point, that is, the maximum value happens at [tex]x_{v}[/tex], and it's value is [tex]y_{v}[/tex].

y = -2x2 + 8x + 5

Quadratic function with [tex]a = -2, b = 8, c = 5[/tex]

What is the maximum height that the stone reaches?

y value of the vertex. So

[tex]\Delta = 8^2-4(-2)(5) = 64 + 40 = 104[/tex]

[tex]y_{v} = -\frac{104}{4(-2)} = 26[/tex]

The maximum height that the stone reaches is of 26 feet.

Find two integers whose sum is 5 and product is -66 TWO INTEGRES
NOT 1 TWO

Answers

Answer:

-6 and 11

Step-by-step explanation:

The only factors of 66 are 11, 6, 2 and 3

11*6 is the only pair whose product is 66

The difference betwen 11 and 6 is 5

11 must be positive and 6 negative since the sum is positve

Please help I’ll give brainliest

Answers

Answer:

33

Step-by-step explanation:

The formula for solving the area of a Trapezoid is a+b/2*h if we substitute the value to 16+3/2*3 it will equal 33

Which one is it?
a) <
b) =
c) >

Answers

Answer:

A: less that

Step-by-step explanation:

4( 14)+3

36+3= 39

39 (2+9)6-7

(11)6-7

66-7

59

39<59

ABCD is a trapezium in which AB ∥ CD, AD ⊥ DC, AB = 4 cm, BP = 7 cm and DC = 25 cm, then area of the trapezium is 45.

Answers

Question:

ABCD is a trapezium in which AB ∥ CD, AD ⊥ DC, AB = 4 cm, BP = 7 cm and DC = 25 cm, then area of the trapezium is ____.

Answer:

101.5cm²

Step-by-step explanation:

The figure for the trapezium ABCD is attached to this response.

To find the area of the trapezium, we use the formula for calculating the area of a trapezium given as follows;

A  = [tex]\frac{1}{2}[/tex] x (a + b) x h            ------------------(i)

Where;

A = area of the trapezium

a and b are the parallel sides of the trapezium

h = height of the trapezium.

As shown in the figure,

AB and CD are parallel

Therefore, we can say that;

a = AB

b = CD

Also, from the figure,

The height of the trapezium is BP

Therefore, we can say that;

h = BP

We know that;

a = AB = 4cm

b = CD = 25cm

h = BP = 7cm

Substitute these values of a, b and h into equation (i) as follows;

A  = [tex]\frac{1}{2}[/tex] x (4 + 25) x 7    

A  = [tex]\frac{1}{2}[/tex] x (29) x 7    

A = 101.5

Therefore, the area of the trapezium is 101.5cm²

Which side lengths do not form a right triangle?
a. 5, 12, 13
b. 10, 24, 28
c. 15, 36, 39
d. 50, 120, 130

Answers

Answer:

B.

Step-by-step explanation:

28 is not a multiple of 13.

factorize: 9x^2 - 24xy + 16y^2

Answers

9x²-24xy+16y²

(3x)²-2.(3x)(4y) +(4y)²

(3x-4y)²

(3x-4y)(3x-4y)

Answer:

[tex](3x -4y)^2[/tex]

Step-by-step explanation:

[tex]9x^2 - 24xy + 16y^2 = (3x)^2 - 24xy + (4y)^2[/tex]

                           [tex]=(3x)^2 -2(3x \times 4y) + (4y)^2\\[/tex]

a = 3x , b = 4y . The expression [tex]a^2 - 2ab + b^2 = (a-b)(a-b) =(a-b)^2[/tex]

                           [tex]=(3x -4y)(3x - 4y)[/tex]

[tex]9x^2 - 24xy + 16y^2 = (3x -4y)^2[/tex]

                           

The function C(x) = 400x - 0.2x ^ 2 represents the total costs for a company to produce a product, where in dollars and x is the number of units sold. Which statement is true? is the total cost

Answers

Answer: it b y’all

Step-by-step explanation: I took the test

Answer:

B

Step-by-step explanation:

To determine the maxima and minima of the polynomial, differentiate the given based on x and equate to 0.                                   C(x) = 400x - 0.2x²                               dC(x) / dt = 400 - 0.4 x = 0 The value of x is 1000. This is the value of the maxima. As the value of C(x) continously becomes lesser as the value of x is set higher, the minima is not identified. Substitute x to the original equation,                               C(x) = (400)(1000) - 0.2(1000²) = $ 200,000Thus, the answer is letter B. 

John starts on the ground floor of a building. He runs up five flights of stairs, then runs
up 6 flights of stairs, then runs down 8 flights of stairs, then runs up four flights of stairs,
and then stops. Each story of the building is 11 feet high. How many feet above the
ground is John when he stops? PLS HELP

Answers

Answer:

77 feet above the ground.

Step-by-step explanation:

After going up 11 flights, (5 +6), he goes down 8, which brings you to 3 and then back up to 7 (3 +4). Since each story is 11 feet high, you multiply 7 and 11, to get a total of 77.

Which expression is equivalent to
(3m² + 5m - 2) + (-2m +18)?
Someone answer ASAP

Answers

3m^2 +5m-2-2m+18

3m^2 +3m+16

Answer:

3m² + 3 m + 16

Step-by-step explanation:

( 3 m ² + 5 m - 2 ) + ( - 2m + 18 )

Remove the unnecessary parantheses and rewrite

3 m ² + 5 m - 2 - 2 m + 18

combine like terms

3m² + 5 m - 2 m + 18 - 2

calculate the sum

3m² + 3 m + 16

ahhh help its PLATO
'la

Answers

Answer:

D

Step-by-step explanation:

To find the inverse of a function, first replace f(x) with y.

Next, switch all the x's and y's, and then solve for y.

Finally, replace y with f-1(x).

Use the following function rule to find f(8). f(x) = (-4 + x)2
f(8) =​

Answers

Answer:

16

Step-by-step explanation:

f(x) = (-4 + x)^2

Let x=8

f(8) = (-4 + 8)^2

Parentheses  first

     = 4^2

Then powers

    = 16

Find the equation of the line for each situation.

a line with a y - intercept of 4 and a slope of -1
a line passing through the point (3, 4) and having a slope of -2
a line passing through the points (1, -1) and (9, 3)

Answers

Step-by-step explanation:

1) y = -x+4

2) y-4= -2(x-3)

y = -2x+6+4

y = -2x+10

3) slope = (3+1)/(9-1) = 4/8=½

the Equation =>

y+1= ½(x-1)

y = ½x -½-1

y = ½x - 1½ or 2y = x -3

I will give 25 points for it if solve this problem ​

Answers

Answer:

5/27

30/81 = 10/17

it's just ratios but if you need then vice versa, swap numerator and denominator

Answer:

the person on top is correct :)

Step-by-step explanation:


If you rolled two dice, what is the probability
that you would roll a sum of 12?
Give your answer as a simplified fraction, Enter
the number that belongs in the green box.
second
first
1 2 4 5 6
11,1 1,2 1,3 1,4 1,5 1,6
212,1 2,2 2,3 2,4 2,5 2,6
33,1 3,2 3,3 3,4 3,5 3,6
44,1 4,2 4,3 4,4 4,5 4,6
5 5,1 5,2 5,3 5,4 5,5 5,6
6 6,1 6,2 6,3 6,4 6,5 6,6

Answers

Probability is 1/36.

What is probability?

The probability is defined as the possibility of an event to happen is equal to the ratio of the number of outcomes and the total number of outcomes.

Two dices were rolled.

Total outcomes in sample space: S = 36

Outcomes which have sum as 12 are E = {(6,6)}

Required Probability: P(E)= 1/36

Hence, Probability is 1/36 of which have sum as 12

Learn more about Probability

https://brainly.com/question/14210034

#SPJ2

Answer:

1/36

Step-by-step explanation:

What is 8 weeks from June 30th

Answers

Answer:

The answer is25th August

A teacher buys a bag of chips and a candy bar for each of the 25 students in her class. Each bag of chips cost $0.40. The teacher spends as total of $65 for chips and candy bars

Answers

Answer: 2.20 per candy bar

Step-by-step explanation:

1. 0.40(25) + x(25) = 65

2. 10 + 25x = 65

3. 25x = 55

4. x = 2.20

2
А
B
G
30°
F
D
NOTE: Angles not necessarily drawn to scale.

Answers

Answer:

x = 60°

Step-by-step explanation:

∠ CGE = ∠ FGD = 30° ( vertical angles )

∠ CGB = 90° , then

x + 30° = 90° ( subtract 30° from both sides )

x = 60°

What is the missing number? I’ll mark brainly

Answers

Answer:

I think it is 4

I am not sure about it

What’s the line of y= -1/2x+2

Answers

Answer:

A picture of the graph is attached

Step-by-step explanation:

I need this ASAP. Thanks​

Answers

Answer:

-6

Step-by-step explanation:

since x = - 5 is < 3

for x < 3,

f(x) = 3x + 9

placing the value of x as - 5

f(x) = 3× (-5) + 9

= -15 + 9

= - 6

Calculate the sizes of the unknown angles in each diagram.

Please, answer with explanation.

Answers

Step-by-step explanation:

a) 2z+3z+4z=180

9z=180

z=20

So,three angles are 40,60 and 80

b) (y+30)+(2y+80)+(y+10)=180

y+30+2y+80+y+10=180

4y+120=180

4y=60

y=15

Three angles are 45,110,25

Find the angle for this problem

Answers

Answer:

ans: 31°

Step-by-step explanation:

<EFG = <EFH + <HFG

2x+52= 21 + ( x+31)

2x-x= 21 +31 -52

x= 0

so our question is to find <HFG so,

x+31°= 31°

Please help I need it!!!

Answers

Answer:

I think it is the first one i think

Step-by-step explanation:

Please answer correctly !!!!! Will mark Brianliest !!!!!!!!!!!!!!

Answers

Answer:

Step-by-step explanation:

the formula is C=2√πA

so simplifying that would give us c=2(4π)

simplify again and we get c=8π

Other Questions
The area of the rectangular floor in Tamara's room is 95 5/6 squarefeet. The width of the room is 8 1/3 feet. What is the length of Tamara'sroom? The storage space in a moving truck is shaped like a rectangular prism. It has a total volume of 16 cubic meters. The height and width are both 2 meters less than the depth. What are the dimensions of the storage space? Which statement accurately describes the difference between short-term and long-term capital gains in terms of taxes? A. Long-term capital gains are from investments that have been held for more than one year and are taxed at a lower rate than short-term capital gains. B. Long-term capital gains are from investments that have been held for at least six months and are taxed at a lower rate than short-term capital gains. C. Long-term capital gains are from investments that have been held for more than one year and are taxed at a higher rate than short-term capital gains. D. Long-term capital gains are from investments that have been held for at least six months and are taxed at a higher rate than short-term capital gains. Audrey is appointed as a research director at a well-known pharmaceutical company. She is asked to design a set of research studies that will test the effectiveness of the drugs that are being manufactured by the company. To ensure that experimenter bias and research participant bias do not influence the results, Audrey should design a set of Read the excerpt from Ovids "Pyramus and Thisbe".Pyramus had lefta little later than his Thisbe had,and he could see what surely were the tracksof a wild beast left clearly on deep dust.His face grew ashen. And when he had foundthe bloodstained shawl, he cried: "Now this same nightwill see two lovers lose their lives: she wasthe one more worthy of long life: it's Iwho bear the guilt for this.Which statement best describes how the order of events creates tension? Tyrone's bowling scores for the first four games of the season were 120,123,105, and 109. He wants to increase his mean score to 125. What score does Tyrone need in the fifth game so that his mean score is 125? Explain your reasoning or show all of your work. Choose all the scenarios below that would cause a decrease in the supply of labor in the agricultural industry.a. More women are entering the workforce in all industries in the United States. b. As the population in the United States ages, fewer workers have the physical ability to work in the industry. c. As the economy expands, so does the need for janitors and cleaning staff employees, causing the wage for nonskilled workers in the cleaning industry to rise. d. Due to global warming, working conditions on farms have become undesirable for more and more people. e. The adoption of strict immigration laws reduces the number of legal and illegal immigrants in the United States. f. There is an increase in the use of biofuels in the United States, which drives up the demand for corn and soybean crops. g. The government passed a new plan to provide government-funded health care for all workers in the agricultural industry. h. New machinery is replacing workers in the harvesting of crops. the rows from left to right in the periodic table is called what Darrell builds and sells bookcases to earn money. He earns $20 for each bookcase he sells. The more bookcases he sells, the more money he earns. What is the dependent variable in this situation? 96 sq meters 144 sq meters84 sq meters102 sq metersPls show work I get different answers from people every time Jaideep changes $12 000 into euros () to buy land in another country.The exchange rate is 1 = $1.33.Calculate the number of euros Jaideep receives.A farmer sold food grains for 9200 rupees through an agent. The rate of commission was 2%. How much amount did the agent get?The price of a machine depreciates at 10% every year. If the present value of the machine be Rs.100000, what will be its value after 3 years?Lucinda invests $500 at a rate of 5% per year simple interest. Calculate the simple interest Lucinda has after 3 years.PLEASE GIVE ANSWERS TO THESE QUESTIONS REALLY QUICK Tim has 5 yellow and 10 blue marbles in a bag. What isthe probability of choosing a yellow marble from thebag? Which of the following processes transfers carbon directly from sea water to sea plants Think for a moment about driving down the street alone in your car when all of sudden you see a car flipped over in the middle of the street, the car is leaking some sort of fluid, you cant tell what it is yet but it doesnt look good and the driver is alone and not moving. What would you do? For every action or forcein nature, there is an equaland opposite reaction. Find the vertex: y=1/2 x^2+2x-3Please explain how to find the answer too! Divide (16m-10m+2m)2m 497100. -----------------. 1037 How does bacteria cause disease? Fill in the coefficients that will balance the following reaction: